Skip to content

Commit

Permalink
docs: 大修第 6 至 8 章的讲解框架 (#87)
Browse files Browse the repository at this point in the history
* 向预备知识一节添加了多元笛卡尔积并修改了一些记号

* 修改部分格式问题

* 将 `\le`, `\ge` 替换为 `\leqslant`, `\geqslant`
为第二章线性空间去除一条定理的情况添加了例子并作为习题

* 修改了矩阵和矩阵表示的定义,引入了交换图,并修改了若干typo和格式问题

* colonV -> colon V

* 修改 ch5 一处表达问题

* 修改 ch1 一处表达问题

* 重写了线性映射矩阵表示、矩阵运算和过渡矩阵、相抵标准型等内容,修改了一些已知typo

* 修改了一些表述

* 修改了语法错误

* 修改对行列向量记号的解释

* 修改自然同构的说法

* 修改 ch7 一处表述问题

* 细化可逆矩阵消去律的描述

---------

Co-authored-by: yhwu-is <[email protected]>
  • Loading branch information
gtj1 and yhwu-is authored Sep 29, 2024
1 parent 888fd01 commit c027c03
Show file tree
Hide file tree
Showing 8 changed files with 720 additions and 286 deletions.
2 changes: 1 addition & 1 deletion 讲义/专题/16 相似标准形:动机与基础.tex
Original file line number Diff line number Diff line change
Expand Up @@ -11,7 +11,7 @@ \section{相似的定义与性质}
我们希望$A$``比较简单''的,就像相抵标准形那样,有尽可能多的$0$,非零元素的排列规律也尽可能简单.

如果从矩阵的角度出发,很自然地,我们会希望与上面映射的目标建立联系,我们会想起任意矩阵$A$都是线性变换$\sigma(\alpha)=A\alpha$在自然基下的表示矩阵. 此时我们的目标可以与映射的目标产生联系,即我们希望找到另一组基,使得$\sigma$在这组基下的矩阵$B$比较简单. 此时$A$$B$的关系是同一个线性变换$\sigma$在不同基下的表示矩阵,事实上这一关系我们其实并不陌生,早在相抵标准形一讲中我们就提到如下定理:
\begin{theorem}{基的选择对映射矩阵的影响}{}
\begin{theorem}{基的选择对变换矩阵的影响}{}
设线性变换$\sigma \in \mathcal{L}(V,V)$$B_1=\{\alpha_1,\ldots,\alpha_n\}$$B_2=\{\beta_1,\ldots,\beta_n\}$是线性空间的$V(\mathbf{F})$的两组基,基$B_1$变为基$B_2$的变换矩阵为$C$,如果$\sigma$在基$B_1$下的矩阵为$A$,则$\sigma$关于基$B_2$所对应的矩阵为$C^{-1}AC$.
\end{theorem}
这一定理研究了同一个线性变换在不同基下表示矩阵之间的关系,契合了我们的需求. 我们可以将上述同一线性变换在不同基下的矩阵表示之间的关系称为\term{相似}. 当然这是从线性映射的角度出发的定义,有时我们谈到相似可能仅仅是矩阵层面的关系,因此我们给出直接的使用矩阵定义如下:
Expand Down
2 changes: 1 addition & 1 deletion 讲义/专题/2 线性空间.tex
Original file line number Diff line number Diff line change
Expand Up @@ -188,7 +188,7 @@ \section{线性空间的定义}

\item 右分配律

(笔者暂时没有想到更好的例子,所以这个例子会略微抽象)考虑 $F=\mathbf{C}, V=\mathbf{C}^2$. 保持 $\mathbb{C}^2$ 上的加法不变,对 $k\in\mathbf{C}, \alpha = (z_1, z_2)$ 定义乘法
(笔者暂时没有想到更好的例子,所以这个例子会略微抽象)考虑 $F=\mathbf{C}, V=\mathbf{C}^2$. 保持 $\mathbf{C}^2$ 上的加法不变,对 $k\in\mathbf{C}, \alpha = (z_1, z_2)$ 定义乘法
\[
k\cdot\alpha = \begin{cases}
(k z_1, k z_2), & z_1 = 0~\text{或}~\Im(z_2 / z_1) \geqslant 0\\
Expand Down
4 changes: 2 additions & 2 deletions 讲义/专题/3 有限维线性空间.tex
Original file line number Diff line number Diff line change
Expand Up @@ -469,8 +469,8 @@ \subsection{极大线性无关组的求法}
对其进行倍加变换,即将第 $i$ 行的 $k$ 倍加到第 $j$ 行上,即第 $j$ 行变为 $a_{j1} + ka_{i1}, a_{j2} + ka_{i2}, \ldots, a_{jn} + ka_{in}$. 记原先的列向量为 $S_A = \{\alpha_1, \alpha_2, \ldots, \alpha_n\}$,新的列向量为 $S_B = \{\beta_1, \beta_2, \ldots, \beta_n\}$.

\begin{enumerate}
\item $S_A$ 线性相关 $\Rightarrow$ $S_B$ 线性相关:若 $S_A$ 线性相关,则存在不全为零的数 $x_1, x_2, \ldots, x_n$ 使得 $x_1\alpha_1 + x_2\alpha_2 + \cdots + x_n\alpha_n = 0$. 由于 $S_B$$S_A$ 仅有第 $j$ 行不同,所以我们仅需要判断第 $j$ 行在系数 $x_1, x_2, \ldots, x_n$ 下的线性组合是否为 0 即可. 事实上我们有 $x_1(a_{j1} + ka_{i1}) + x_2(a_{j2} + ka_{i2}) + \cdots + x_n(a_{jn} + ka_{in}) = x_1a_{j1} + x_2a_{j2} + \cdots + x_na_{jn} + k(x_1a_{i1} + x_2a_{i2} + \cdots + x_na_{in}) = 0$,故 $S_B$ 线性相关.
\item $S_A$ 线性无关 $\Rightarrow$ $S_B$ 线性无关:若 $S_A$ 线性无关,使用反证法,假设 $S_B$ 线性相关,但我们知道$S_A$ 实际上就是 $S_B$ 的第 $j$ 行减去第 $i$ 行的 $k$ 倍,因此 $S_B$ 变换到 $S_A$ 可以视为做了一个倍加变换,所以根据证明的第 1 点可知 $S_A$ 线性相关,出现矛盾.
\item $S_A$ 线性相关 $\implies$ $S_B$ 线性相关:若 $S_A$ 线性相关,则存在不全为零的数 $x_1, x_2, \ldots, x_n$ 使得 $x_1\alpha_1 + x_2\alpha_2 + \cdots + x_n\alpha_n = 0$. 由于 $S_B$$S_A$ 仅有第 $j$ 行不同,所以我们仅需要判断第 $j$ 行在系数 $x_1, x_2, \ldots, x_n$ 下的线性组合是否为 0 即可. 事实上我们有 $x_1(a_{j1} + ka_{i1}) + x_2(a_{j2} + ka_{i2}) + \cdots + x_n(a_{jn} + ka_{in}) = x_1a_{j1} + x_2a_{j2} + \cdots + x_na_{jn} + k(x_1a_{i1} + x_2a_{i2} + \cdots + x_na_{in}) = 0$,故 $S_B$ 线性相关.
\item $S_A$ 线性无关 $\implies$ $S_B$ 线性无关:若 $S_A$ 线性无关,使用反证法,假设 $S_B$ 线性相关,但我们知道$S_A$ 实际上就是 $S_B$ 的第 $j$ 行减去第 $i$ 行的 $k$ 倍,因此 $S_B$ 变换到 $S_A$ 可以视为做了一个倍加变换,所以根据证明的第 1 点可知 $S_A$ 线性相关,出现矛盾.
\end{enumerate}
\end{proof}

Expand Down
8 changes: 4 additions & 4 deletions 讲义/专题/4 线性空间的运算.tex
Original file line number Diff line number Diff line change
Expand Up @@ -267,17 +267,17 @@ \section{线性空间的直和}

定理的证明是基本的.
\begin{proof}
上述命题等价,只需证明 (1) $\Rightarrow$ (2) $\Rightarrow$ (3) $\Rightarrow$ (4) $\Rightarrow$ (1).
上述命题等价,只需证明 (1) $\implies$ (2) $\implies$ (3) $\implies$ (4) $\implies$ (1).

先证 (1) $\Rightarrow$ (2): 设 $W_1 + W_2$ 中的 $\alpha$ 有两个分解式
先证 (1) $\implies$ (2): 设 $W_1 + W_2$ 中的 $\alpha$ 有两个分解式
$$
\alpha = \alpha_1 + \alpha_2 = \beta_1 + \beta_2, \quad \alpha_1, \beta_1 \in W_1, \ \alpha_2, \beta_2 \in W_2,
$$
$\alpha_1 - \beta_1 = \beta_2 - \alpha_2 \in W_1 \cap W_2 = \{\mathbf{0}\}$,于是得 $\beta_1 = \alpha_1, \beta_2 = \alpha_2$,故 $\alpha$ 的分解式是唯一的。

其次证 (2) $\Rightarrow$ (3): 由 $W_1 + W_2$ 中零向量的分解式唯一性以及 $0 = 0 + 0$,立即得命题 (3) 成立.
其次证 (2) $\implies$ (3): 由 $W_1 + W_2$ 中零向量的分解式唯一性以及 $0 = 0 + 0$,立即得命题 (3) 成立.

再证 (3) $\Rightarrow$ (4): 由命题 (3) 可推出 $W_1 \cap W_2 = \{\mathbf{0}\}$,因为若 $W_1 \cap W_2 \ne \{\mathbf{0}\}$,则存在 $0 \ne a \in W_1 \cap W_2$,使得 $0 = \alpha + (-\alpha)$,(其中 $\alpha \in W_1, -\alpha \in W_2$),这与命题 (3) 相矛盾。再根据维数公式 就得命题 (4)。
再证 (3) $\implies$ (4): 由命题 (3) 可推出 $W_1 \cap W_2 = \{\mathbf{0}\}$,因为若 $W_1 \cap W_2 \ne \{\mathbf{0}\}$,则存在 $0 \ne a \in W_1 \cap W_2$,使得 $0 = \alpha + (-\alpha)$,(其中 $\alpha \in W_1, -\alpha \in W_2$),这与命题 (3) 相矛盾。再根据维数公式 就得命题 (4)。

最后由命题 (4) 及维数公式 立即得命题 (1) 成立.
\end{proof}
Expand Down
62 changes: 45 additions & 17 deletions 讲义/专题/5 线性映射.tex
Original file line number Diff line number Diff line change
Expand Up @@ -437,24 +437,41 @@ \section{线性映射的像与核}
\end{enumerate}

\begin{example}{}{}
已知$\mathbf{R}^3$$\mathbf{R}^2$的映射$\sigma$$\sigma(x_1,x_2,x_3)=(x_1+x_2,x_2-x_3)$,求$\sigma$的像和核.
已知$\mathbf{R}^3$$\mathbf{R}^2$的映射$\sigma$$\sigma(x_1,x_2,x_3)^{\color{lightgray}\mathrm{T}}=(x_1+x_2,x_2-x_3)^{\color{lightgray}\mathrm{T}}$,求$\sigma$的像和核.
\end{example}
注:这里需要对记号进行一个澄清,事实上 $\mathbf{R}^n$ 空间中的向量都应当是列向量,但为了节省空间,一些教材在描述映射的情形下会写成行向量的形式. 但笔者认为线性映射相关的地方行和列的混乱容易导致一些困惑,为此,笔者有时会在向量的右上角加上一个浅色的转置 $({}^{\color{gray}\mathrm{T}})$ 以代表它是列向量.

\begin{solution}
\begin{itemize}
\item 首先求像空间. 取出发空间$\mathbf{R}^3$的一组基$B=\{(1,0,0),(0,1,0),(0,0,1)\}$,则$\im \sigma=\sigma(\mathbf{R}^3)=\spa(\sigma(1,0,0),\sigma(0,1,0),\sigma(0,0,1))=\spa((1,0),(1,1),(0,-1))$. 根据求解极大线性无关组的方法(或者这么简单的情况瞪眼法也可以)得到像空间$\im \sigma=\spa((1,0),(0,-1))=\mathbf{R}^2$.

\item 接下来求解核空间. 设$\sigma(\alpha)=0$,其中$\alpha=(x_1,x_2,x_3)$,即$\sigma(x_1,x_2,x_3)=(x_1+x_2,x_2-x_3)=(0,0)$,解得解向量为$k(-1,1,1),\enspace k\in\mathbf{R}$,写成线性扩张的形式为$\spa((-1,1,1))$.
\item 首先求像空间. 取出发空间$\mathbf{R}^3$的一组基$B=\{(1,0,0)^{\color{lightgray}\mathrm{T}},(0,1,0)^{\color{lightgray}\mathrm{T}},(0,0,1)^{\color{lightgray}\mathrm{T}}\}$,则
\begin{align*}
\im \sigma
&=\sigma(\mathbf{R}^3)
&=\spa(
\sigma(1,0,0)^{\color{lightgray}\mathrm{T}},
\sigma(0,1,0)^{\color{lightgray}\mathrm{T}},
\sigma(0,0,1)^{\color{lightgray}\mathrm{T}}
) \\
&=\spa(
(1,0)^{\color{lightgray}\mathrm{T}},
(1,1)^{\color{lightgray}\mathrm{T}},
(0,-1)^{\color{lightgray}\mathrm{T}}
)
\end{align*}
根据求解极大线性无关组的方法(或者这么简单的情况瞪眼法也可以)得到像空间
$\im \sigma=\spa((1,0)^{\color{lightgray}\mathrm{T}},(0,-1)^{\color{lightgray}\mathrm{T}})=\mathbf{R}^2$.

\item 接下来求解核空间. 设$\sigma(\alpha)=0$,其中$\alpha=(x_1,x_2,x_3)^{\color{lightgray}\mathrm{T}}$,即$\sigma(x_1,x_2,x_3)^{\color{lightgray}\mathrm{T}}=(x_1+x_2,x_2-x_3)^{\color{lightgray}\mathrm{T}}=(0,0)$,解得解向量为$k(-1,1,1)^{\color{lightgray}\mathrm{T}},\enspace k\in\mathbf{R}$,写成线性扩张的形式为$\spa((-1,1,1)^{\color{lightgray}\mathrm{T}})$.
\end{itemize}
\end{solution}

下面我们也给出另一种求像空间的方法,但是为了防止读者混淆这一方法和之后线性映射矩阵表示的方法,希望读者能按照笔者首先介绍的方法求解.
\begin{solution}\label{ex:线性映射的像空间求解2}
\begin{align*}
\sigma(x_1, x_2, x_3) &= (x_1 + x_2, x_2 - x_3)\\
&= x_1(1, 0) + x_2(1, 1) + x_3(0, -1)
\sigma(x_1, x_2, x_3)^{\color{lightgray}\mathrm{T}} &= (x_1 + x_2, x_2 - x_3)^{\color{lightgray}\mathrm{T}}\\
&= x_1(1, 0)^{\color{lightgray}\mathrm{T}} + x_2(1, 1)^{\color{lightgray}\mathrm{T}} + x_3(0, -1)^{\color{lightgray}\mathrm{T}}
\end{align*}
$e_1 = (1, 0), e_2 = (1, 1), e_3 = (0, -1)$,则 $\sigma(x_1, x_2, x_3) = x_1 e_1 + x_2 e_2 + x_3 e_3$,所以 $\sigma(\mathbf{R}^3) = \spa\{e_1, e_2,e_3\} = \spa \{e_1, e_2\}=\mathbf{R}^2$.
$\beta_1 = (1, 0)^{\color{lightgray}\mathrm{T}}, \beta_2 = (1, 1)^{\color{lightgray}\mathrm{T}}, \beta_3 = (0, -1)^{\color{lightgray}\mathrm{T}}$,则 $\sigma(x_1, x_2, x_3)^{\color{lightgray}\mathrm{T}} = x_1 \beta_1 + x_2 \beta_2 + x_3 \beta_3$,所以 $\sigma(\mathbf{R}^3) = \spa\{\beta_1, \beta_2, \beta_3\} = \spa \{\beta_1, \beta_2\}=\mathbf{R}^2$.
\end{solution}


Expand Down Expand Up @@ -517,7 +534,7 @@ \section{线性映射的确定}
\end{proof}
最后我们讨论一个初学时容易困惑的问题,如下例所示:
\begin{example}{}{线性映射判断1}
是否存在$\mathbf{R}^2$$\mathbf{R}^3$的线性映射$\sigma$使得$\sigma(1,0)=(1,0,0),\enspace\sigma(0,1)=(0,1,0),\enspace\sigma(1,1)=(0,0,1)$
是否存在$\mathbf{R}^2$$\mathbf{R}^3$的线性映射$\sigma$使得$\sigma(1,0)^{\color{lightgray}\mathrm{T}}=(1,0,0)^{\color{lightgray}\mathrm{T}},\enspace\sigma(0,1)^{\color{lightgray}\mathrm{T}}=(0,1,0)^{\color{lightgray}\mathrm{T}},\enspace\sigma(1,1)^{\color{lightgray}\mathrm{T}}=(0,0,1)^{\color{lightgray}\mathrm{T}}$
\end{example}

初学时感到困难是因为不能熟练应用线性映射的各类性质,找不到映射定义也不敢下结论不存在,或者发现必要条件都满足了却不敢构造. 我们这里给出几个解决策略:
Expand All @@ -536,29 +553,40 @@ \section{线性映射的确定}
\item 如果题目给定的映射不违反上述线性映射的必要条件,那我们可以按照\autoref{thm:线性映射构造} 构造出相应的映射.
\end{enumerate}

根据上面的描述,我们发现 \ref*{item:5:线性映射判断1:1} 中的映射定义违反了明显违反了不能是从低维到高维满射的条件. 实际上,$\sigma$也将线性相关的向量组映射到了线性无关的向量组,并且根据定义,$\sigma(0)=\sigma((1,0)+(0,1)-(1,1))=(1,1,-1)$,因此所有的必要条件都被违反了,因此这一映射一定不是线性映射. 事实上这一例子也表明很多时候三个必要条件可能是同时违反的,因为它们都是由基本的线性映射和线性相关性质推导而来,并非完全独立的判据.
根据上面的描述,我们发现 \ref*{item:5:线性映射判断1:1} 中的映射定义违反了明显违反了不能是从低维到高维满射的条件. 实际上,$\sigma$也将线性相关的向量组映射到了线性无关的向量组,并且根据定义,$\sigma(0)=\sigma((1,0)^{\color{lightgray}\mathrm{T}}+(0,1)^{\color{lightgray}\mathrm{T}}-(1,1)^{\color{lightgray}\mathrm{T}})=(1,1,-1)$,因此所有的必要条件都被违反了,因此这一映射一定不是线性映射. 事实上这一例子也表明很多时候三个必要条件可能是同时违反的,因为它们都是由基本的线性映射和线性相关性质推导而来,并非完全独立的判据.

我们还需强调的是,如果\autoref{thm:线性映射构造} 前提成立,即题目给我们的是一组基下的像,则一定不会违反上述三个必要条件. 对于条件 \ref*{ex:线性映射判断1},给定一组基$\alpha_1,\ldots,\alpha_n$,我们要凑出$\sigma(0)$只能通过$\sigma(0)=\sigma(0\alpha_1+\cdots+0\alpha_n)=0\sigma(\alpha_1)+\cdots+0\sigma(\alpha_n)=0$,因此不可能违反 \ref*{item:5:线性映射判断1:1}. 对于 \ref*{item:5:线性映射判断1:2},我们给定的是基,因此不存在将线性相关向量组映射到线性无关向量组的情况. 对于 \ref*{item:5:线性映射判断1:3},如果题目给出的是低维到高维的映射,由于我们只给出了低维出发空间的基下的像,这些像不可能张成整个高维到达空间(原理和$n-1$个向量无法张成$n$维空间一致),因此也不可能违反 \ref*{item:5:线性映射判断1:3},因此\autoref{thm:线性映射构造} 并不与我们的必要条件相矛盾,相反,如果题目给出的是一组基下的像,我们就可以毫无顾虑地说映射一定存在.

最后我们再看一个例子来练习我们上面的策略:
\begin{example}{}{线性映射判断2}
是否存在$\mathbf{R}^3$$\mathbf{R}^2$的线性映射$\sigma$使得$\sigma(1,-1,1)=(1,0),\enspace\sigma(1,1,1)=(0,1)$
是否存在$\mathbf{R}^3$$\mathbf{R}^2$的线性映射$\sigma$使得$\sigma(1,-1,1)^{\color{lightgray}\mathrm{T}}=(1,0)^{\color{lightgray}\mathrm{T}},\enspace\sigma(1,1,1)^{\color{lightgray}\mathrm{T}}=(0,1)^{\color{lightgray}\mathrm{T}}$
\end{example}

\begin{solution}
事实上这里的定义完全不违反上述的必要条件,因此我们考虑证明这一线性映射存在. 根据\autoref{thm:线性映射构造},我们考虑构造出$\sigma$在一组基(我们取自然基)下的像,然后我们就可以根据\autoref*{thm:线性映射构造} 知道这一映射一定存在.

实际上,根据提给条件我们有
\begin{gather*}
\sigma(1,-1,1)=\sigma(e_1-e_2+e_3)=\sigma(e_1)-\sigma(e_2)+\sigma(e_3)=(1,0) \\
\sigma(1,1,1)=\sigma(e_1+e_2+e_3)=\sigma(e_1)+\sigma(e_2)+\sigma(e_3)=(0,1)
\sigma(1,-1,1)^{\color{lightgray}\mathrm{T}}
=\sigma(e_1-e_2+e_3)
=\sigma(e_1)-\sigma(e_2)+\sigma(e_3)
=(1,0)^{\color{lightgray}\mathrm{T}} \\
\sigma(1,1,1)^{\color{lightgray}\mathrm{T}}
=\sigma(e_1+e_2+e_3)
=\sigma(e_1)+\sigma(e_2)+\sigma(e_3)
=(0,1)^{\color{lightgray}\mathrm{T}}
\end{gather*}
我们希望解出$\sigma(e_1),\sigma(e_2),\sigma(e_3)$,这样就可以直接根据\autoref*{thm:线性映射构造} 构造出这一线性映射. 但这一方程组只有2个方程却有三个未知量. 事实上我们可以任意定义$\sigma(e_3)=(0,0)$,然后解方程组得到
\[\sigma(e_1)=\dfrac{1}{2} (1,1),\enspace \sigma(e_2)=\dfrac{1}{2} (-1,1)\]
又由$\sigma(e_3)=(0,0)$,根据\autoref*{thm:线性映射构造},满足题目条件的线性映射存在.
\[\sigma(e_1)=\dfrac{1}{2} (1,1)^{\color{lightgray}\mathrm{T}},\enspace \sigma(e_2)=\dfrac{1}{2} (-1,1)^{\color{lightgray}\mathrm{T}}\]
又由$\sigma(e_3)=(0,0)^{\color{lightgray}\mathrm{T}}$,根据\autoref*{thm:线性映射构造},满足题目条件的线性映射存在.
\end{solution}
需要注意的是题目中$\sigma(e_3)$不一定要定义为$(0,0)$,这样只是为了计算方便,事实上定义成任何值都可以得到$\sigma$在一组基下的像,从而根据\autoref*{thm:线性映射构造} 得到这一线性映射存在. 如果题目要求我们写出映射也并不复杂,根据我们在\autoref*{thm:线性映射构造} 中的构造方法,我们可以写出$\forall\alpha=(x,y,z)=xe_1+ye_2+ze_3$
\[\sigma(\alpha)=\sigma(xe_1+ye_2+ze_3)=x\sigma(e_1)+y\sigma(e_2)+z\sigma(e_3)=\dfrac{1}{2} (x-y,x+y)\]
\[
\sigma(\alpha)
=\sigma(xe_1+ye_2+ze_3)
=x\sigma(e_1)+y\sigma(e_2)+z\sigma(e_3)
=\dfrac{1}{2} (x-y,x+y)^{\color{lightgray}\mathrm{T}}
\]
符合题目条件. 事实上根据$\sigma(e_3)$定义的不唯一,我们可以得到不同的线性映射,这里只是给出一种可能的解.

\section{线性映射基本定理}
Expand Down Expand Up @@ -1178,10 +1206,10 @@ \subsection{自然同构} \label{subsec:自然同构}
\item$V_1,V_2,V_3$分别为$m,n,s$维线性空间,$\sigma\in \mathcal{L}(V_1,V_2),\enspace\tau\in \mathcal{L}(V_2,V_3)$,则
\[r(\sigma)+r(\tau)-n \leqslant r(\tau\sigma) \leqslant \min\{r(\sigma),r(\tau)\}.\]

\item$V_1$是有线维线性空间$\sigma,\tau\in \mathcal{L}(V_1,V_2)$,则
\item$V_1$是有限维线性空间$\sigma,\tau\in \mathcal{L}(V_1,V_2)$,则
\[r(\sigma+\tau) \leqslant r(\sigma)+r(\tau).\]

事实上前两题的结论在下一章节矩阵的秩中都会涉及,此处有兴趣的同学可以尝试从线性映射的角度理解这两个秩不等式. 如果无法找到合适方式,可以考虑化为矩阵进行证明.
事实上前两题的结论在后续章节矩阵的秩中都会涉及,此处有兴趣的同学可以尝试从线性映射的角度理解这两个秩不等式. 如果无法找到合适方式,可以考虑化为矩阵进行证明.

\item$\sigma\in \mathcal{L}(V,V)$$\dim V_1=n$,且$\sigma^2=\sigma$$I$$V$上的恒等变换. 证明:
\begin{enumerate}
Expand Down
Loading

0 comments on commit c027c03

Please sign in to comment.